2020 AMC 12A Problems/Problem 17

Revision as of 21:07, 1 February 2020 by Lopkiloinm (talk | contribs) (Solution 1)

Problem 17

The vertices of a quadrilateral lie on the graph of $y=\ln{x}$, and the $x$-coordinates of these vertices are consecutive positive integers. The area of the quadrilateral is $\ln{\frac{91}{90}}$. What is the $x$-coordinate of the leftmost vertex?

$\textbf{(A) } 6 \qquad \textbf{(B) } 7 \qquad \textbf{(C) } 10 \qquad \textbf{(D) } 12 \qquad \textbf{(E) } 13$

Solution 1

Realize that ln$\frac{91}{90}$ is extremely small, so the $x$-coordinates must be very close to each other. Also realize that $\frac{91}{90} = \frac{182}{180} = \frac{13*14}{12*15}.$ $12$ is the smallest number there, and therefore must be the answer. ~lopkiloinm